LSAT and Law School Admissions Forum

Get expert LSAT preparation and law school admissions advice from PowerScore Test Preparation.

User avatar
 Dave Killoran
PowerScore Staff
  • PowerScore Staff
  • Posts: 5853
  • Joined: Mar 25, 2011
|
#27275
Complete Question Explanation

Flaw in the Reasoning—#%. The correct answer choice is (B)

The conclusion of the argument states that the rumored declines in automobile-industry revenues are exaggerated (a numerical statement), but the premises provided in support of this argument only address the market share percentages of the three groups that have automobile-industry revenues (percentage statements). The percentage statements used by the author only indicates that the percentages have changed, not whether overall revenue has changed:
Oct 92_LR2_#24 powerscore_lsat_course_M12_L9_explanations_Numbers_#6_diagram_1.png
Although the composition of the market has changed in terms of the market share of each group, this fact tells us nothing about industry revenues because market shares will always add up to 100% regardless of the actual dollars involved. Thus, automobile-industry revenues could have fallen dramatically and the percentages above could still be accurate.

Answer choice (A): Although it is true that the possibility is left open that the statistics for the manufacturers’ share may come from a different source, this does not address the fundamental percentage-to-number error in the argument.

Answer choice (B): This is the correct answer choice. The answer reveals the error of the author: the changing market shares of different groups have no impact on the actual amount of revenues. In all instances, the market shares will add up to 100%, so a discussion of shifts within this 100% is meaningless as far as making a determination of whether revenues declined.

Answer choice (C): This is not a flaw of the argument. The author is allowed to simply note that the shares changed and use those facts to draw a conclusion. In the argument the conclusion is faulty, but not for the reason cited in this answer.

Answer choice (D): The interrelationship of the groups named in the stimulus is not an issue in determining whether the conclusion is in error.

Answer choice (E): The argument is about revenues, and information about profits will not describe the error in the reasoning.
You do not have the required permissions to view the files attached to this post.
 Nikki Siclunov
PowerScore Staff
  • PowerScore Staff
  • Posts: 1362
  • Joined: Aug 02, 2011
|
#23815
The conclusion states that the rumored declines in auto-industry revenues are exaggerated, i.e. that the revenues haven't declined as much as people seem to believe. This is a numerical statement. The premises, however, only address the shift in market shares of the three groups that comprise the auto-industry. These are all percentage statements: they indicate how the shares of the revenue have changed, but give us no indication as to how the overall revenue has changed. In other words, the fact that some pieces in the pie have expanded and others have contracted relative to each other tells us nothing about the overall size of the pie. Market shares will always add up to 100% regardless of the actual dollars involved. This is precisely what answer choice (B) states.

Answer choice (A): There is no reason to suspect that the statistics from either source are biased, so this issue has no bearing on the conclusion.

Answer choice (B): This is the correct answer choice. Given that all market shares add up to 100%, a discussion of how the different shares have reallocated is meaningless as far as overall revenues are concerned.

Answer choice (C): This is true, but irrelevant. The author is under no obligation to clarify why the shares have changed, because the argument is not causal.

Answer choice (D): The interdependence of the groups named has no bearing on the argument's conclusion.

Answer choice (E): The argument is about revenues, not profits. Although it is technically correct - revenues are not the only determinant of profits - this claim has no bearing on the conclusion of the argument.
 Katya W
  • Posts: 42
  • Joined: Dec 03, 2019
|
#75150
Hi Nikki, can I get further explanation of why the author is not obligated to explain why the revenue shares of different sectors of the industry changed? Just because the argument is not causal, I don’t see why it is not a flaw for him to base his conclusion on missing information. It is also confusing to me that I had to choose between two answers that were “true”. Could this also be explained? How do I choose the between two “true” answers? And could someone better explain to me what is the difference between Flaw of Reasoning and Flaw of Argument?

Thank you so much!

Katya
 Adam Tyson
PowerScore Staff
  • PowerScore Staff
  • Posts: 5153
  • Joined: Apr 14, 2011
|
#75252
Hey there Katya! Just because an answer is true doesn't mean that it is a Flaw. Here's an example:

My grandmother's blueberry pie is better than the pie I bought at the bakery because it tastes better and has a better mouth feel.

It's true that I did not consider whether my wife's pie is also good. But that's not a flaw in my argument, because I wasn't arguing about my wife's pie, but only about the comparison between my grandmother's and the one from the bakery. That true statement is not a flaw because it is not important to the argument.

Now for an example that is closer to the one in this stimulus:

Last year I spent $1000 on coffee at Starbucks, this year I spent $1000 on coffee at Coffee Bean. Therefore, I spent the same amount on coffee this year as last year.

The flaw is that I failed to consider that in either or both of the years examined, I may have spent money on coffee at other places besides the ones I named.

It's true that I didn't say anything about WHY I switched coffee chains, or even IF I switched chains. But that is not relevant to the argument that is strictly about how much I spent. I don't need to tell you why it happened, I just need to give evidence that supports my claim that it DID happen. "Why" simply doesn't matter! That's true of the stimulus, too - we don't need to know why the percentages shifted, we just need to know whether those shifting percentages prove the conclusion or not.

Flaw in the Reasoning is the term we use for these questions, but Flaw in the Argument would mean the exact same thing.
 Katya W
  • Posts: 42
  • Joined: Dec 03, 2019
|
#75261
Hi Adam,

Thank you! I think that helped! Flaws can be hard for me to wrap my head around, and numbers and percentages are like a foreign language to me written in chicken scratch. My brain simply cannot digest the numbers and percentages topic. I have never been adept at math or numbers. Including market share. Actually, are you aware of any resources where I could specifically further practice implementing the numbers and percentages lessons? I am working on the practice drills in the Workbook, but I feel more would do me well. For some reason I struggle to visualize numbers vs percentages scenarios, but I know if I have more practice I can figure it out. It's just a block.

Thank you Adam!!
User avatar
 Stephanie Oswalt
PowerScore Staff
  • PowerScore Staff
  • Posts: 811
  • Joined: Jan 11, 2016
|
#75272
Katya W wrote: Actually, are you aware of any resources where I could specifically further practice implementing the numbers and percentages lessons? I am working on the practice drills in the Workbook, but I feel more would do me well.
Hi Katya,

Thanks for the reply. Keep up the hard work :D! You can find some additional numbers and percentages questions available for digital purchase here: PowerScore Digital LSAT Testing and Analytics Package.

Thanks!
 mackenziemarcus
  • Posts: 3
  • Joined: Jan 22, 2021
|
#85697
How can we make the assumption that manufacturers, suppliers, and service companies are the only 3 sectors of this market?
User avatar
 Dave Killoran
PowerScore Staff
  • PowerScore Staff
  • Posts: 5853
  • Joined: Mar 25, 2011
|
#85700
mackenziemarcus wrote: Fri Mar 19, 2021 3:16 pm How can we make the assumption that manufacturers, suppliers, and service companies are the only 3 sectors of this market?
Because the numbers add up to 100%. In other words, the question itself indicates there can be only these three sources, because otherwise it would add up to less than 100% of the "automobile-industry revenues." With all three adding up to 100%, they take up the entire market. Tricky, but we don't make an assumption here; the facts prove it to be the case as stated by the author.

Does that help?

Get the most out of your LSAT Prep Plus subscription.

Analyze and track your performance with our Testing and Analytics Package.